Đến nội dung

davildark nội dung

Có 224 mục bởi davildark (Tìm giới hạn từ 19-04-2020)



Sắp theo                Sắp xếp  

#473740 Tìm số lớn nhất thỏa

Đã gửi bởi davildark on 29-12-2013 - 18:49 trong Tổ hợp và rời rạc

Từ một nhóm  25 người chúng ta thành lập một số nhóm nhỏ. mỗi nhóm nhỏ có đúng 5 thành viên và hai nhóm nhỏ bất kì có tối đa một thành viên chung. Hãy tìm số lớn nhất có thể của các nhóm nhỏ?

 




#410935 Đề thi olympic 30/4 lớp 10 miền Nam 2012-2013

Đã gửi bởi davildark on 06-04-2013 - 23:30 trong Thi HSG cấp Tỉnh, Thành phố. Olympic 30-4. Đề thi và kiểm tra đội tuyển các cấp.

Sặc đề sai rùi $y-1$ không phải $1-y$




#410911 Đề thi olympic 30/4 lớp 10 miền Nam 2012-2013

Đã gửi bởi davildark on 06-04-2013 - 22:12 trong Thi HSG cấp Tỉnh, Thành phố. Olympic 30-4. Đề thi và kiểm tra đội tuyển các cấp.

Toạ độ trọng tâm tam giác là $x_{G}=\frac{x_{A}+x_{B}+x_{C}}{3}$

 

Và $y_{G}=\frac{y_{A}+y_{B}+y_{C}}{3}$ nên ta phải chỉ ra trong 19 cặp $(x_{i};y_{i})$ (i=1 đến 19) tồn tại 3 cặp thoả mãn

 

$3|x_{m}+x_{n}+x_{p}$

 

và $3|y_{m}+y_{n}+y_{p}$   (***)

 

Thật vậy.Trong 19 số $x_{i}$ tồn tại ít nhất 7 số $x_{1},x_{2},...,x_{7}$ đồng dư modun 3

 

xét 7 số $y_{1},y_{2},...,y_{7}$ cũng có it nhất 3 số $y_{a},y_{b},y_{c}$ đồng dư modun 3  (theo nguyên lí dirichlet)

 

Khi đó 3 cặp số $(x_{a};y_{a}),(x_{b};y_{b}),(x_{c};y_{c})$ thoả mãn  (***)

 

Hay đây là 3 đỉnh của một tam giác có trọng tâm nguyên

Dễ vậy mà làm không được :((

Thui làm câu pt hàm vậy 

Dễ dàng quy nạp được $f(n) \leq n$ với $n\equiv 1 mod 3$

Từ đó ta có $2014\geq f(2014)\geq f(2)+2012$

$\Rightarrow f(2) \leq 2$

Quy nạp 1 lần nữa ta có  $f(n) \leq n$ với $n\equiv 2 mod 3$

$\Rightarrow 2015 \geq f(2015) \geq f(3)+2012$

$\Rightarrow f(3) \leq 3 $

Từ đó dễ dàng ta có $f(2013) \leq 2013$

Mà $f(2013) \geq 2013 $

Vậy $f(2013)=2013$

 

Àh câu 2 yêu cầu tìm diện tích lục giác câu này khoai nhất @@




#409453 $\sum \frac{1}{\sqrt{ab}}...

Đã gửi bởi davildark on 31-03-2013 - 15:49 trong Bất đẳng thức và cực trị

Cho $ a,b,c > 0 $ và $ ab+bc+ac=3 $ Chứng minh:

$\sum \frac{1}{\sqrt{ab}}\geq \sum a$




#404286 [MO2013] - trận 22 - PT, BPT, HPT, HBPT

Đã gửi bởi davildark on 11-03-2013 - 22:13 trong Thi giải toán Marathon dành cho học sinh Chuyên Toán 2013

Kết luận sai nghiệm :( ẩu quá



#403875 [MO2013] - trận 22 - PT, BPT, HPT, HBPT

Đã gửi bởi davildark on 10-03-2013 - 21:43 trong Thi giải toán Marathon dành cho học sinh Chuyên Toán 2013

Đặt $\left\{\begin{matrix}
x=\tan A & \\
y=\tan B & \\
z=\tan C &
\end{matrix}\right. $ Với $ A,B,C \in \left [ -\frac{\pi }{2} ,\frac{\pi }{2} \right ]$
Điều kiện $\cos A,B,C \neq 0$
Khi đó $x+y+z=xyz$
$$\Leftrightarrow \tan A +\tan B + \tan C =\tan A \tan B \tan C$$
$$\Leftrightarrow -\tan A=\tan (B+C) \\ \Leftrightarrow A+B+C=k\pi $$
Do điều kiện của A,B,C nên $ A+B+C=\pi $
Xét $\sin 2A=0$
$ \Rightarrow \sin A =0$
$ \Rightarrow \tan A=0 \\ \Rightarrow x=0$
$ \Rightarrow x=y=z=0 $
Thử lại $x=y=z=0$ là 1 bộ nghiệm
Xét $ \sin A,B,C \neq 0$
Ta có $\frac{156x}{x^{2}+1}=78\sin 2A$
Tương tự ta có $156\sin 2A=65\sin 2B=60\sin 2C$
Ta có $2A=2\pi -2B-2C \Rightarrow \sin 2A=\sin (2\pi -2B-2C)=-\sin (2B+2C)$
$$\Rightarrow 156\sin 2A=-156(\sin 2B\cos 2C + \sin 2C\cos 2B) $$
$$\Rightarrow 65\sin 2B=-156\sin 2B\cos 2C -169\sin 2B\cos 2C \ ( do \sin 2C=\frac{13}{12}\sin 2B) $$
$$\Rightarrow 65 + 165\cos 2C +169\cos 2C =0$$

SAI
Tương tự ta có hệ phương trình
$$\left\{\begin{matrix}
65+156\cos 2C + 169\cos 2B=0\\
60+65\cos 2A +25\cos 2C=0 \\
156+60\cos 2B + 144\cos 2A =0
\end{matrix}\right.$$
Giải ta có
$$\left\{\begin{matrix}
\cos 2A =-\frac{12}{13}\\
\cos 2B= -\frac{5}{13}\\
\cos 2C= 0
\end{matrix}\right.$$
$$\Leftrightarrow \left\{\begin{matrix}
x^2=25\\
y^2=\frac{9}{4}\\
z^2=0
\end{matrix}\right.$$
Thử lại ta có các bộ nghiệm $(x,y,z) = (0,0,0) , (5,$\frac{3}{2}$,0) ,(-5,$\frac{-3}{2}$,0)$

 

Điểm bài: 3




#400346 [MO2013] - Trận 20 - Tổ hợp và rời rạc

Đã gửi bởi davildark on 27-02-2013 - 10:51 trong Thi giải toán Marathon dành cho học sinh Chuyên Toán 2013

Mình thử thì hình như các số 101 , 1001 , 10101 vẫn thoả điều kiện cua số x



#394110 $CMR: \frac{1}{MA}+\frac{1}...

Đã gửi bởi davildark on 06-02-2013 - 22:18 trong Hình học

Cho 3 đường tròn $(O),(T),(W)$ Trong đó $(T),(W)$ tiếp xúc trong với $ (O)$ tại $B,C$ và $(T)(W)$ tiếp xúc nhau tại $M$. Tiếp tuyến chung,trong của $(T),(W)$ cắt $(O)$ tại $A,N$,Đường thẳng $A,B$ cắt $(T)$ tại điểm $P$, Đường thẳng $AC$ Cắt $(W)$ tại $Q$. $CMR: \frac{1}{MA}+\frac{1}{MN}=\frac{2}{PQ}$

Bài hình TPHCM vòng 2 2012-2013 :D



#391805 Cho tam giác ABC có 3 điểm $A(5;1), B(-3;3), C(4;7)$, $M(2;3)...

Đã gửi bởi davildark on 30-01-2013 - 19:20 trong Hình học phẳng

Mình nói hướng làm nhé
Gọi tọa độ vecto pháp tuyến của d là $\overrightarrow{n_{d}}=(A,B)$
Dễ dàng viết pt AB AC
Do tam giác ABC cân tại A Từ đó viết pt góc giải được A B



#384764 Thông báo 1 : Khóa học "Soạn thảo tài liệu khoa học với $\LaTeX...

Đã gửi bởi davildark on 08-01-2013 - 19:06 trong Nơi diễn ra Khóa học

Em cũng đăng ký :D



#384257 [MO2013] - Trận 16 Đa thức, phương trình hàm

Đã gửi bởi davildark on 06-01-2013 - 21:04 trong Thi giải toán Marathon dành cho học sinh Chuyên Toán 2013

Lời giải

Nếu $f(y_{1})=f(y_{2}) \\ \Rightarrow (f(x))^2+y_{1}=(f(x))^2+y_{2} \\ \Rightarrow y_{1}=y_{2}$
Nên f là 1 đơn ánh
VP của (1) là 1 hàm bậc nhất theo y nên nhận tập trị trên $\mathbb{R}$ nên f nhận giá trị trên $\mathbb{R}$ nên f là 1 toàn ánh
Vậy f là song ánh
$\Rightarrow \exists \ a $ để $f(a)=0$
Trong (1) thay $x=y=a$ ta có $ f(0)=a$
Trong(1) thay $x=0$ thì $f(f(y))=(f(0))^2+y$ (2)
Trong (2) thay $y=a$ thì $ a = a^2 +a \Rightarrow a=0$
Vậy $f(0)=0$
Trong (1) thay y=0 ta có $f(xf(x))=(f(x))^2 $ (3)
Lại từ (2) ta thay y=0 thì $f(f(x))=x$
Trong (3) thay $x=f(x)$ ta có $ f(xf(x))=x^2$ (4)
Từ (3) và (4) $\Rightarrow (f(x))^2=x^2$
$\Rightarrow \left\{\begin{matrix}
f(x)=x \\
f(x)=-x
\end{matrix}\right.$
Thử lại $f(x)=x$ và $f(x)=-x$ thỏa đề bài
Vậy $f(x)=x$ và $f(x)=-x$ là 2 hàm số cần tìm
___________________________________
Điểm bài làm: $d=10$

S = 3 + 10*3 = 33



#380962 ĐỀ THI CHUYỂN HỆ KÌ I MÔN TOÁN-LỚP 10

Đã gửi bởi davildark on 27-12-2012 - 19:44 trong Thi tốt nghiệp

Còn mỗi câu hình chém lun :D
Hình đã gửi

a) Gọi E là điểm giữa cung nhỏ BC dễ dàng chứng minh E là trung điểm $II_{a}$
Ta có $OI=OK$ và $OM=ON$
$\Rightarrow KE // MI $
Mà $ KE \perp QI_{a} $
$\Rightarrow MI \perp QI_{a}$
Mà $\widehat{IQI_{a}}=90^{\circ} \Rightarrow IQ \perp QI_{a}$
Vậy M , I , Q thẳng hàng
b) Gọi $F=QM \cap I_{a}K$ và $S=AM\cap QI_{a}$
Ta có $\widehat{FQI_{a}}=90^{\circ}$
Nên F là điểm đối xứng với $I_{a}$ qua K $\Rightarrow =\widehat{I_{a}NF}=90^{\circ}$
Xét $\bigtriangleup FII_{a}$ có $MK // II_{a} $ và K là trung điểm $FI_{a}$
$\Rightarrow MI=MF $
$\Rightarrow NF//IP \Rightarrow \widehat{NPI}=\widehat{IPI_{a}}=90^{\circ}$
$\Rightarrow IP \perp MI_{a} $
Xét $\bigtriangleup MSI_{a}$ có I là trực tâm nên $SI \perp MI_{a} $
Vậy S , I , P thẳng hàng (dpcm)
____________________

DXT:Chả hiểu sao lúc thi mình vẽ hình bài này tới 3 lần mà toàn sai! Chán chả buồn vẽ lại nữa nên bỏ nguyên câu hình!@#$



#380514 Đề thi chuyển hệ kì I THPT chuyên ĐHSPHN

Đã gửi bởi davildark on 25-12-2012 - 23:32 trong Thi HSG cấp Tỉnh, Thành phố. Olympic 30-4. Đề thi và kiểm tra đội tuyển các cấp.

Sang đây thảo luận :D
http://diendantoanho...962#entry380962



#379860 $\frac{1}{2x+y+6}+\frac{1}{...

Đã gửi bởi davildark on 23-12-2012 - 17:43 trong Bất đẳng thức và cực trị

Đặt $x=2a$ $y=2b$ $z=2c $
$\Rightarrow abc=1 $
Áp dụng AM-GM và đẳng thức này
$\frac{1}{a+ab+1}+\frac{1}{b+bc+1}+\frac{1}{c+ca+1}=1$ với $abc=1$
Ta có
$VT=\frac{1}{2}(\sum \frac{1}{2a+b+3})\leq \frac{1}{2}\sum \frac{1}{2\sqrt{ab}+2\sqrt{a}+1}=\frac{1}{4}$



#377942 $-x^4 + 2mx^2 + m^2 -2 = 0 $

Đã gửi bởi davildark on 15-12-2012 - 23:14 trong Phương trình - hệ phương trình - bất phương trình

Cho phương trình $-x^4 + 2mx^2 + m^2 -2 = 0 $
Tìm m để pt có 4 nghiệm phân biệt $x_{1} , x_{2} , x_{3} , x_{4} $ thoả $x_{1}-x_{2}=x_{2}-x_{3}=x_{3}-x_{4}$



#371986 Chứng minh NB là tiếp tuyến

Đã gửi bởi davildark on 23-11-2012 - 22:56 trong Hình học

Cho M ngoài (O) Vẽ 2 tiếp tuyến MA MC và đường kính AB .Đường thẳng vuông góc với AB tại O cắt BC tại E
a) CM OM//BE
b) Vẽ CH vuồng AB CF vuông AM OM cắt AC tại I CM F I H thẳng hàng
c) K là trung điểm CH AK giao MC tại N . CM NB là tiếp tuyến của (O)
d) CM B K M thẳng hàng



#371690 I,K,L thẳng hàng

Đã gửi bởi davildark on 22-11-2012 - 23:11 trong Hình học

Được bạn à, Thoải mái. Cám ơn bạn

Hình đã gửi
Không biết đúng không nữa
Gọi $J'=BD\cap BN$
Áp dụng định lý Papus cho 2 bộ ba điểm thẳng hàng A,P,B và C,D,N với
$Q=AD \cap DC \\ J'=BD \cap PN \\ M=AN \cap BC$
$\Rightarrow Q , J' , M $ thẳng hàng
$\Rightarrow J'= QM \cap BN $
$\Rightarrow J'\equiv J $ hay B , D , J thẳng hàng

Áp dụng định lý Desargues cho $\bigtriangleup QDN$ và $\bigtriangleup MBP $ với
$K=QD \cap MB \\ L=DN \cap BP \\ I=QN \cap MP$
Mà theo phần trên ta có QM BD NP đồng quy tại J
Nên ta có I , K , L thẳng hàng



#371671 I,K,L thẳng hàng

Đã gửi bởi davildark on 22-11-2012 - 22:35 trong Hình học

Bài toán:
Cho tứ giác ABCD, M, N thuộc CB,CD sao cho A,M,N thẳng hàng.Các điểm P,Q thuộc AB,AD sao cho C,P,Q thẳng thàng. AD giao CB tại K, AB giao CD tại L, MP giao NQ tại I, MQ giao NP tại J. Chứng minh rằng:
a) I,K,L thẳng hàng
b) J,B,D thẳng hàng

Bài này mình làm câu b rùi mới làm câu a có được không :D



#365998 $$a^3+b^3+c^3+5abc+4\geq 4(ab+bc+ca)$$

Đã gửi bởi davildark on 30-10-2012 - 19:56 trong Bất đẳng thức - Cực trị

Bài toán 1.[Đề thi cuối kì trường chuyên TB 2011]
Ch0 các số thực dương $a,b,c$.Chứng minh rằng:
$$(a+b+c)\left(\frac{1}{a}+\frac{1}{b}+\frac{1}{c}\right)\geq \frac{4(a+b+c)^2}{ab+bc+ca}-3$$
Bài toán 2.
Ch0 các số thực không âm $a,b,c$.Chứng minh bất đẳng thức sau:
$$a^3+b^3+c^3+5abc+4\geq 4(ab+bc+ca)$$

Bài 1 Viết lại bdt như sau
$\frac{a}{b}+\frac{a}{c}+\frac{b}{c}+\frac{b}{a}+\frac{c}{a}+\frac{c}{b}+6\geq \frac{4(a+b+c)^2}{ab+bc+ac}$
$\Leftrightarrow \frac{(a+b)^2}{ab}+\frac{(b+c)^2}{bc}+\frac{(a+c)^2}{ac}\geq \frac{4(a+b+c)^2}{ab+bc+ac}$
Đúng theo Cauchy-Schwarz
Bài 2 Ta dùng dồn biến
Đặt $f(a,b,c)=a^3+b^3+c^3+5abc+4-4(ab+bc+ac)$
Ta sẽ CM $f(a,b,c)\geq f(a,\sqrt{bc},\sqrt{bc})\geq 0$
Xét $f(a,b,c)-f(a,\sqrt{bc},\sqrt{bc}) \\ =b^3+c^3-4a(b+c)-2bc\sqrt{bc}+2a\sqrt{bc} \\ =(\sqrt{b^3}-\sqrt{c^3})^2-4a(b+c-2\sqrt{bc}) \\ =(\sqrt{b}-\sqrt{c})^2(b+\sqrt{bc}+c)^2-4a(\sqrt{b}-\sqrt{c})^2 \\ =(\sqrt{b}-\sqrt{c})^2[((b+\sqrt{bc}+c)^2)-4a]$
Tới đây giả sử $a=min{a,b,c}$ thì $f(a,b,c)\geq f(a,\sqrt{bc},\sqrt{bc})$
Ta cm tiếp $f(a,\sqrt{bc},\sqrt{bc})\geq 0$
$a^3+2bc\sqrt{bc}+5abc+4\geq 8a\sqrt{bc}+4bc$
Tới đây tịt ngòi . Thử = máy tính thấy đúng mà CM chưa được



#365646 $f(x)=xf(\frac{1}{x})$

Đã gửi bởi davildark on 28-10-2012 - 21:52 trong Phương trình hàm

Em thì làm theo cách khác
Cho $x=y$ ta có $2f(x)=1+f(2x)$
$f(2x)=2xf(\frac{1}{2x})=x.2f(\frac{1}{2x})=x(1+f(\frac{1}{x}))=x+xf(\frac{1}{x})=x+f(x)$
$f(2x)=f(x+x)=f(x)+f(x)-1=2f(x)+1$
$\Rightarrow 2f(x)-1=f(x)+x $
$\Rightarrow f(x)=x+1$



#364867 $f(x)=xf(\frac{1}{x})$

Đã gửi bởi davildark on 25-10-2012 - 23:03 trong Phương trình hàm

Tìm các hàm số $f : \mathbb{R}\rightarrow \mathbb{R}$ đồng thời thỏa mãn các điều kiện sau
$f(x)=xf(\frac{1}{x})$
$f(x)+f(y)=1+f(x+y)$



#364747 Đề thi HSG 12 tỉnh Bình Định năm 2013

Đã gửi bởi davildark on 25-10-2012 - 18:42 trong Thi HSG cấp Tỉnh, Thành phố. Olympic 30-4. Đề thi và kiểm tra đội tuyển các cấp.

Bài 5 (2,0 đ).

Tìm tất cả các hàm số $f: \mathbb{R} \to \mathbb{R}$ thỏa:

$i)$ $f(0)=\frac{1}{2}$
$ii)$ Với mọi $x,y \in \mathbb{R}$ tồn tại $a \in \mathbb{R}$ sao cho:$ f(x+y)=f(x)f(a-y)+f(y)f(a-x)$.


Cho $x=y=0$ ta có $\frac{1}{2}=\frac{1}{2}f(a)+\frac{1}{2}f(a)\Rightarrow f(a)=\frac{1}{2}$
$y=0$ thì $f(x)=f(a-x)$
$\Rightarrow f(x+y)=f(x)f(a-y)+f(y)f(a-x)=2f(x)f(y)$
$f(a)=f(a-x+x)=2f(a-x)f(x)=2(f(x))^2\Rightarrow (f(x))^2=\frac{1}{4}\Rightarrow f(x)=\frac{1}{2}$



#364326 Tìm quỹ tích điểm $M$

Đã gửi bởi davildark on 23-10-2012 - 22:11 trong Hình học phẳng

Hình đã gửi
$F=DI \ \cap \(O) $ nên F cố định
E là trung điểm KF
Dễ dàng CM K là trung điểm BC và $IF=FC$
$\Rightarrow EM=\frac{IF}{2}=\frac{FC}{2}$
Vậy M di chuyển trên đường tròn tâm E bán kính $\frac{FC}{2}$



#363449 [MO 2013] Trận 9 - Hình học

Đã gửi bởi davildark on 20-10-2012 - 23:52 trong Thi giải toán Marathon dành cho học sinh Chuyên Toán 2013

Hình đã gửi
Trong bài làm em xin đặt lại các điểm cho dễ làm $M_{1}=M$ $M_{2}=N$
Gọi I là trung điểm MN
Vẽ đường tròn đi qua O , A và tiếp xúc với $d_{1}$ . ĐƯờng tròn này cắt $d_{2}$ tại B
Tương tự C là giao điểm của đường tròn qua O , A tiếp xúc với $d_{2}$ và $d_{1}$
Gọi D là trung điểm OB . E là trung điểm OC
Vì B,C cố định nên D,E cũng cố định . Ta sẽ CM I nằm trên đường DE cố định
Thật vậy
Ta có $\widehat{AMN}=\widehat{AON}=\widehat{ABO}$ và $\widehat{ANM}=\widehat{AOB}$
$\Rightarrow \bigtriangleup AMN\sim \bigtriangleup ABO$
$\Rightarrow \frac{MN}{OB}=\frac{AM}{AB}\Rightarrow \frac{IM}{BD}=\frac{AM}{AB} \Rightarrow \bigtriangleup AIM \sim \bigtriangleup ADB$
$\Rightarrow \widehat{AIM}=\widehat{ADB}=\widehat{ADM}$
Vậy tứ giác AIDM nội tiếp
Tương tự ta cũng có tứ giác AINE nội tiếp . Khi đó
$\widehat{AIE}=\widehat{ANE}=\widehat{AMO}=\widehat{AMD}$
Mà $\widehat{AMD}+\widehat{DIA}=180^{\circ}$
$\Rightarrow \widehat{AIE}+\widehat{DIA}=180^{\circ}$
$\Rightarrow$ D,I,E thẳng hàng
Vậy I di chuyển trên DE cố định

Nên dùng góc định hướng để tổng quát hơn.
D-B=27.9h
E=9
F=0
S=39



#361907 [MO2013] Trận 8 - PT, BPT, HPT, HBPT

Đã gửi bởi davildark on 14-10-2012 - 21:31 trong Thi giải toán Marathon dành cho học sinh Chuyên Toán 2013

Nhờ trọng tài del bài trước hộ em bài trước em làm sai

Bài giải

Theo khai triển newton ta có
$(a+b)^x=\sum_{k=0}^{x}\binom{n}{k}a^{x-k}b^k$
Cho $a=1$ $b=-1$ ta có
$\binom{x}{0}-\binom{x}{1}+\binom{x}{2}-\binom{x}{3}+...+(-1)^x\binom{x}{x}=0$
Mặt khác ta có
$$1=\binom{x}{0} \\ x=\binom{x}{1}\\\frac{x(x-1)}{1.2}=\binom{x}{2}\\
\\
... \\
\\
\frac{x(x-1)...(x-n+1)}{n!}=\binom{x}{n}
$$
Từ đó đề bài tương đương với
$$\binom{x}{0}-\binom{x}{1}+\binom{x}{2}-\binom{x}{3}+...+(-1)^n\binom{x}{n}=\binom{x}{0}-\binom{x}{1}+\binom{x}{2}-\binom{x}{3}+...+(-1)^x\binom{x}{x}$$
$\Rightarrow x=n$
Vậy pt có 1 nghiệm là $x=n$

S=0